Anda di halaman 1dari 72

Problem 3.

1:
The Bright-star Firm pays $ 108,000 for a piece of heavy equipment. The
equipment has a life of 12 years with no salvage value. After 6 years of use, an
overhaul costing $10,800 will be needed. If money can be invested at a nominal rate
of 15%, how much must the firm deduct from the equipment yield by the end of each
year over its 12-year life in order to recover the capital and all other costs associated
with the equipment?

SOLUTION
Capital money for equipment =108000$
n =12%
i =15%
Over haul cost =10800$ after 6 years of use

F1

F2
A=?

0 1 2 3 4 5 6 7 8 9 10 11 12

P0 P6

General solution:
F/P i, n
F1=P0 (
And since the formula of the factor is (1+i)n
Numerical
solution:
F/P 15, 12
F1=$108000( ) = $108000*(5.35025) =$577827.01
F/P 15, 6
F2=$10800( ) = $10800*(2.31061) = $24981.056
F=F1+F2=$602808.06
General solution:
A/F i, n
A=F ( ) and the formula of the factor is given by: i/ [(1+i)n -1]
The numerical solution will be:
A/F 15, 12

1
A=602808.06( ) = 602808.06*(0.0344807) =$ 20,785.24
Problem 3.1:
The bright star firm pays $108,000 for apiece of heavy equipment .the
equipment has a life of 12 years with no salvage value .after 6 years of use, an
overhaul costing will be needed .if money can be invested at nominal interest of
15%, how much must the firm deduct from the equipment yield
By the end of each year over its 12-year life in order to recover the capital and all
other costs associated with the equipment?

Solution 3.1:
The cash flow diagram

F1

F0
A A AA A A A A A A A

1 2 3 4 5 6 7 8 9 10 11 12

P0=$10,8000
P0=$10,8000

F0=P0(F/P r,n)=$108,000(F/P 15 ,12)


(F/P 15, 12)=(1+i)n=(1+0.15)12 = 5.350250
F1=$108,000 X 5.350250=$ 577827.0114
F1=P6(F/P r,n)=$108,00(F/P 15 6)
(F/P 15,6)=(1+i)n=(1+0.15)6 = 2.313060766
F1=10800X2.313060766=$24981.105627
Ft=F0+F1=$24981.105627+$ 577827.0114=$602808.0677
A=F(A/F r,n)= (A/F 15,12)
(A/F 15,12)=0.15/(1+0.15)12 -1=0.034480776
A=F(A/F r,n)= (A/F 15,12)=$602808.0677X0.034480776=$20785.2877

Problem 3.2:
Mr. Mahir borrowed $22,000 and agreed to pay $1390 at the end of each
month for 18 months to discharge the debt . If , Mr. Mahir was unable to pay the tenth
and eleventh installments , what equal monthly payments thereafter would be needed
to discharge the debt on time ?
Solution 3.2

2
Cash flow diagram

A=$1390

1 2 3 4 5 6 7 8 9 18

P9

P  A( P / Ai ,18 )
22000  1390( P / Ai,18 )  ( P / Ai,18 )  15.827338

From tables:

i  1.25%  ( P / Ai,18 )  16.02960


i  1.5%  ( P / Ai,18 )  15.6725

Using interpolation:

i  1.25 1.5  1.25



15.827338  16.02960 15.6725  16.02960
i  1.392%
P9  A( P / Ai,9 )  1390( P / Ai,9 )

From tables:

( P / A1.25,9 )  8.46238
( P / A1.5,9 )  8.36049
x  8.46238 8.36049  8.46238

1.392  1.25 1.5  1.25
 X ( P / A1. 392,9 )  8.4045065
 P9  1390(8.4045065)  $11582.254

3
9 10 11 12 13 14 15 16 17 18

P9 P11

P11= P9 (F/P 1.392 , 2)= P9 (1+i)n


=11682.264 (1+0.01392)2
=$12009.762

i (1  i ) n
1.392 (1.01392) 7
A1 =P11 ( A/P 1.392 , 7
) = P11 ( )=12009.762( 100 )
(1  i ) n  1 7
1.01392  1

=12009.762(0.1509213) =$1812.5297

Problem resolved :

Figure (1) shows the cash flow diagram if Mr. Mahir continues paying the
installments . But he is unable to pay the 10th and 11th payments and the cash flow
diagram will be as in figure (2) . As we see he must pay a higher value deposit
beginning from 12th month until the end of the period .

4
A = $1390

1 2 3 4 5 6 7 8 9 10 11 12 13 14 15 16 17 18

Figure (1)

$22,000

1 2 3 4 5 6 7 8 9 10 11 12 13 14 15 16 17 18

$22000

Finding the interest rate (i) :

A = P(A/P i,n) (n=18)

1390 = 22000(A/P i , n)

(A/P i , n ) = 0.0631818

5
From table (A) by linear interpolation at 18 period we have :

At
( 0A.06238
/P
at ( 0A.0631818
/P
)  i = 1.39017%
1.25,18 i ,18
i = 1.25% )
i = 1.5% ( 0A.06381
/P 1.5,18 )
Finding P , we have (future worth after 9 year):
(1  i ) n  1 (1  0.0139) n  1
Pg = A( F / A 1.39017,9 )=A( )=1390( )
i (i  1) n 0.0139(1  0.0139) n

F / A 1.3917,9 (1  i ) n  1 (1  0.0139) 9  1
P g =F=A( )=A( )=1390( )
i (1  i ) n 0.0139(1  0.0139) 9
P g = $11683.08

Finding the future worth of P g after two year

F = P11 = Pg [(1+i)n] = $11683.08[(1+0.0139)2] = $12010.12

Finding the monthly installments that Mr. Mahir should pay for the last 7 years
. n=7 ,
i = 1.39017%

(1  i ) n  1 (1  0.0139) 7  1
F = P11 = A [ ]  $12010.12 = A [ ]
i (1  i ) n 0.0139(1  0.0139) 7

A = $1812.4

Comment :

The intrinsic difference arise from the two ways of solution (the formula and
the table methods) , which affect the numerical value of the interest rate , that can be
clear from the interest rate values used in the solution above .
To clarify the affection of interest rate value contrast here the percentage error
between the calculated value from the formula and the table value :
Error = [(1.392 – 1.39017)/(1.392)]*100%
= 0.131465%
This lead to a difference between the value of (A) for the two values of (i)
Tolerance of (A) = [(1812.5297 – 1812.4)/(1812.5297)] * 100%
= 0.0071557%

Problem 3.3:
a)How long will it take to discharge a dept of $25,000 using beginning-of-year payments of
$3,800 at a nominal interest rate of 13% compounded continuously?

6
b)If equal payments of $8,500 at the end of each year for 10 years are needed to discharge a
dept of $40,000 , what is the rate of return assuming:
(i)Annual compounding?
(ii)Continuous compounding?

Solution:
a) Cash flow diagram

P/A r, n
P - A  A[ ]
 rn
1 e
25,000  3,800  3,800 r
e 1
0.13 n
1 e
5.5789  0.13
e 1
 0.13n ln 0.22548
e  0.22548  n 
 0.13
 11 .4577 years

 To discharge the dept we need 11.46 years.

b) (i) Cash flow diagram

7
P/A i, n
PA( )

(1i)  1
n

40,000  8,500
i(1 i)
n

Taking i = 16%

(1 0.16)  1
10
P/A 16,10
( )   4.83323
0.16(1 0.16)
10

Taking i = 17%


10

(
P/A 17,10
) 
(1 0. 17) 1
 4.65860
0.17(1 0.17)
10

 The interest rate is 16% < i < 17%. Thus we use linear interpolation

4.83323  4.65860 4.70588  4.65860


  i  16.2707%
17  16 i  16

8
 The interest rate is 16.2707%
b) (ii) Cash flow diagram

P/A i, n
P  A[ ]
 rn
1 e
40,000  8,500 r
e 1
Taking i = 15%
P/A 15,10 1 e
10*0.15

[ ] 0.15
 4.8004
e 1
Taking i = 16%
P/A 16,10 1 e
10*0.16

[ ] 0.16
 4.59973
e 1
 The interest rate is 15% < i < 16%. Thus we use linear interpolation

9
4.8004  4.59973 4.70588  4.59973
  r  15.52897%
16  15 15  r
 The interest rate is 15.52897 %

Problem 3.4:
a) Identify the following interest factors (give their full names ) and use
the interest tables to determine their values. Then check this values using the
interest formulae and determine the percentage error in each case. Are the
percentage errors of the same magnitude? Any comment? (3.2)

i) [ A/ F 7 ,10.3
]
ii) ( P/ A 7.7 , 5.5
)
iii) [ A/G 5.5 , 7.5
]
iv) [ A/ F 10.5 ,13.3
]

b) A man has borrowed $1,500 from the XYZ loan company to by a used
car and has an agreement to repay the loan company $500 at the end of each
month 3-month period and $1,000 at the end of the 9th month. Assuming
quarterly compounding of the interest, determine the nominal and effective
interest rates involved in this agreement.

c) Determine the nominal interest rate, r; a certain amount takes 6 years to


double in magnitude. Assuming;

i) annual compounding
ii ) continuous compounding. (3.2)

SOLUTION:

10
a) [ A / F 7 ,10.3 ] This is an equal-payment-series sinking fund factor for
continuous compounding using tables we get the following value :

[ A/ F 7 ,10.3
] = 0.07152 & [ A/ F 7 ,11
] = 0.06252

Assume:
X 1 = n 1 = 10 & Y 1 = 0.07152
X 2 = n 2 = 11 & Y 2 = 0.06252
X = n 3 = 10.3 & Y = ???????

By linear interpolation :
Y  Y1 X  X1

Y2  Y1 X 2  X 1

Y  0.07152 10.3  10

0.06252  0.07152 11  10

Y=0.06882 ≡ The approximate value of the factor .

By using formulae:
er 1
[ A/ F r ,n
] =
e rn  1
e 0.07  1
[ A/ F 7 ,10.3
] = = 0.06863 ≡ the exact value
e 0.07 (10.3)  1

approximat e value  exact value


percentage error  %
exact value

0.06882  0.06863
 %  0.2768
0.06863

Which is approximately 0.3% of error .

ii. ( P / A 7.7 , 5.5


) This is an equal-payment-series present worth factor

from tables we found :

11
( P/ A 7,5
)  4.10020

(P/ A 8, 6
)  4.62288

(P/ A 8, 5
)  3.99271

(P/ A 8, 6
)  4.62288

By using linear interpolation :

Y  Y1 X  X1

Y2  Y1 X 2  X 1

Solving for

& (P / A ) 7,6
To find ( P / A 7 ,5.5 )
Y  4.1002 5.5  5

4.76654  4.1002 65

Y = 4.43337

And do so for ( P / A 8, 5
) & (P / A 8, 6
) to find ( P / A 8 , 5.5
)

Y  3.99271 0.077  0.07



4.62288  4.43337 0.08  0.07

Y = 4.345467 ≡ approximate value of ( P / A 7.7 , 5.5


)

By using formulae :
 (1  0.077) 5.5  1 
(P / A )  
7.7 , 5.5
5.5 
 0.077(1  0.077) 
 (1  0.077) 5.5  1 
(P / A 7.7 , 5.5
)   5.5  = 3.50809 represent the exact value
 0.077(1  0.077) 
[ A/ G 6,8
]  3.18621
The percentage error is less than -0.2 & this is small error specially in this case
where we use more than one interpolation .

iii) [ A / G 5.5, 7.5 ] this is a uniform-gradiant-series factor for continuous


compounding .
from tables we found :

[ A/G 5, 7
]  2.80042
[ A/G 6, 7
]  2.7607
[ A/G 5 ,8
]  3.23822

12
[ A/ G 6,8
]  3.18621

Using [ A / G 5, 7
] & [ A/ G 5,8
] to find [ A / G 5 , 7 .5
]

Y  Y1 X  X1

Y2  Y1 X 2  X 1

Y  2.80042 7.5  7

3.23822  2.80042 87

Y = 3.01932

And do so for [ A / G 6 , 7 .5
] & [ A/ G 6 ,8
] to find [ A/G 6, 7.5
]
Y  2.7607 7.5  7

3.18621  2.7607 87

Y = 2.973455

Interpolating between [ A / G 6 , 7.5


] & [ A/G 5 , 7 .5
] to find [ A / F 10.5 ,13.3
]:

Y  0.036041 0.105  0.10



0.033222  0.03604 0.11  0.10

Y= 2099638 represents the approximate value.

By using formulae:
0.105
[ A/ G 10.5,13.3
]
e 10.105*13.3

1 0 .055
[ A/G 5.5 , 7.5
]  0.055  0.055*7.5
e 1 e

= 2.997499. represents the exact value

2.99638  2.994799
percentage error  %  0.037%
2.99638
Which is a very small error .

(iv) [ A / F 10.5 ,13.3


] This is a funds-flow sinking fund factor .

By using tables we obtain:

[ A/ F 10 ,13
]  0.03746

13
[ A/ F 11,13
]  0.03461
[ A/ F 11,14
]  0.03002
[ A/ F 11,14
]  0.03002

Using [ A / F 10 ,13.3
] &[ A/ F 10.5 ,13.3
] to find [ A / F 10 ,13.3
]
Y  O.O36041 13.3  13

0.33233  0.036041 14  13

Y= 0.036041 The approximate value

Doing so for [ A / F 11,13.3


] &[ A/ F 10 ,13.3
] to find [ A / F 10.5 ,13.3
]

Y  0.036041 0.105  0.10



0.033222  0.03604 0.11  0.10

Y= 0.034637 approximate value.

By using formulae:
0.105
[ A/ G 10.5,13.3
]
e 1
0.105*13.3

= 0.034528 represents the exact value

0.034637  0.03452
percentage error  %  0.31568%
0.034528
it is approximately small error.

b)
$1500  $500( P / A i , 2 )  $1000( P / F i ,3
)
$1500  $500( P / A i,2
)  $1000( P / F i ,3
)

By trail & error :

At i=13% :right hand side of above equation(R.H.S)=$3.0542


At i= 14% : R.H.S =$2.9966

By interpolating the above:

14%  13% 14%  i



2.9966  3.0542 2.9966  3

i = 0.1394 = 13.94 % quarterly

r = ic = 13.94 % * 4 + 55.76% compounding quarterly

r c 0.5576 4
i eff =(1+ ) -1 = (1+ )  1 = 0.6854
c 4
= 68.54 %

14
i effquar = (1+0.1394) 1 -1= 0.1394 = 13.94 %

c)
i) annual compounding :

F = P[ F / P r , n ]
2P = P( F / P i ,6 )
>>> 2 = (1+i) 6
>>> i = 0.1225 = 12025 %
>>> r = ic = i = 12.25 %

ii) continuously compounding :

F = P [ F / P r ,6 ]
2P = P [ F / P r , 6 ]
2 = e r (6)
r = 0.1155 = 11.55 %

15
Problem 3.5:
a) From the interest table, use linear interpolation to determine the value of
the equal-payment-series capital-recovery factor for 53 periods at 6.25%.

Re-solve the problem using the interest formulae and comment on any
discrepancy between the two values.

b) How long will it take for money to triple if compounded continuously at a


nominal rate of 18%?

c) For funds flowing continuously at a rate of $500 per year with a nominal
interest rate of 8%, sketch the cash flow diagram, then use the interest tables to
determine the minimum period required to accumulate $5000. check your final
answer by using the interest formulae. (3.2)
===========================================================
solution:
A/P 6.25,53

a- Find : ( )
from the table using Interpolation:
A/P 6 , 55 A/P 7 , 50 A/P 6.25,50

(0.06344) (0.07246)  ( )
A/P 6,55 A/P 7,55 A/P 6.25,55

( 0.06254) (0.07147)  ( )
Step1:-
A/P 6.25,50

To Find : ( ) we have:
x1 = i1= 0.06  Y1 = 0.06344

x2 = i2= 0.07  Y2 = 0.07246

x = 0.0625  Y =??

Y-Y1 = Y2-Y1
x-x1 x2-x1

Y – 0.06344 = 0.07246 – 0.06344  Y = 0.065695


0.0625 – 0.06 0.07 – 0.06

Step 2 :-
A/P 6.26,55

To find ( ) we have:

16
x1 = i1 = 0.06  Y1= 0.06254
x2 = i2 = 0.07  Y2 = 0.0717
x = 0.0625  Y = ??

Y – 0.06254 = 0.07174 – 0.06254  Y= 0.06484


0.0625 – 0.06 0.07 – 0.06

b)

F=$3

0 1 2 3 ………………… n
r= 18% comp. continuously

P=$1

F/P 18, n

F=P [ ]
F/P 18, n F/P 18, n

F= [  3= ] [ ]
P 1
Using Reverse Interpolation :-
F/P 18, 6 F/P 18, 7

[ ] [ ]
X 1 = n1 = 6  Y1 = 2.94468

X 2 = n2 = 7  Y2 = 3.52542

X = ??  Y=3

X–6 = 7–6  x = 6.095


3 – 2.94468 3.52542 – 2.94468

n = 6.1 years.
Number of period = 6.1 years.

17
Second Approach using formula:-

[F/P r, n
]=e

F/P r, n
rn
[ ]
= e r n = e 0.18 n
3 = e 0.18 n  0.18 n = Ln 3 = 1.0986  n = 6.1 years .

c)
F = $ 5000 F = $5000
A = $ 500
r =8%
n = ??

0 1 2 ……………… n

F=A [F/A r, n
]
5000 = 500 [F/A r,n
]
A = $ 500

10 = [F/A 8,n
] ( r= 8 %)
nominal interest
i- Using Reverse Interpolation:

F/A 8,7 F/A 8,8

[9.38341] [11.206]
x1=n1 =7  Y1=9.38341

x2= n2 =8  Y2=11.206

x=n  Y=10

x -7 = 8 – 7 x = 7.338 years
10 – 9.38341 11.206 – 9.38341

:. n= 7.7338 years

ii- Using formula:

F = A [e r n – 1]
r
0.08
10 = e –1  1.8 = e 0.08 n
0.08

0.587787 = 0.08n

18
:. n = 7.35 years

Step 3 :-
X1 = n1 = 50  Y1 = 0.065695

X2 = n2 = 55  Y2 = 0.06484

X = 53  Y = ??

Y – 0.065695 = 0.06484 – 0.065695  Y = 0.065182


53 – 50 55 – 50

A/P 6.25, 53

( )
= 0.065182
Using Interest Formula:
(A/P 6.25,53
) = i (1 + i)n = 0.0625 (1.0625)53 .
(1+ i)n – 1 ( 1.0625)53 – 1
A/P 6.25,53
( ) = 0.065119

From the two value we notice that they differ and there is a percentage of error
due to interpolation.

19
Problem 3.6:
a) If an investment of $27000 provides a continuous flow of funds at the rate of
$550 monthly for 5.5 years determine the annual interest rate earned by this
investment assuming continuous compounding.

b) The commercial bank offers the following personal loan plan which is called
'' the six percentage plan'' according to this plan the bank adds 6% to the
amount borrowed; the borrower bays back one-twelfth of this total amount at
the end of each month for a year. If a person borrows $5000 using this plan,
to determine:

i) His monthly payment .


ii) The time interest rate per month
iii) The nominal and effective rates per annum
.
.

Solution:

a)

550  1.06
A  $6600 yearly
12

P / A i, n
PA [ ]

P / A i, n
27000 = 6600 [ ]

P / A i,5.5
4.090909  [ ]
4.090909

From tables – C – by Looking at The present worth Factor columns we see


P / A 11,5 P / A 11,6 [
P / A 11,5.5
]
[ ] [ ]
3.84591 4.39226

20
P / A 12,5 P / A 12,6 [
P / A 12,5.5
]
[ ] [ ]
3.7599 4.27706

By interpolating:-

n  n1 y   y1
 Where Y is present worth factor
n 2  n1 y 2  y1

y = [
P / A 11,5.5
4.119085
]

By interpolation to get i from [ 4P.090909


/ A i,5.5
]

Also
y  = [ 4P.01848
/ A 12,5.5
]

y  y  i1  i2

y  y i  i2

4.119085  4.01848 11  12

4.090909  4.01848 i  12

i  11 .28%

b)

5000  (1.06)
i) A  44.667
12

21
P / A i, n P / A i, n
PA( ) 5000  441.667 ( )

P / A i, n 5000
( )
441.667

P / A i ,n
( )  11.32074617

ii) From the tables:-


P / A 0.75,12
( )  11 .43500

P / A 1,12
( )  11 .25510

by linear interpolation

i  0.75 11 .32074617  11 .43500


 i  0.908%
1  0.75 11 .25510  11 .4300

iii)

r  12  i  12  0.908  10.9%
0.109 12
reff  (1  ) 1
12
 11 .46%

22
Problem 3.7:
a)The blue star company can either buy a certain piece of land and for outdoor
storage of equipment or lease it on a 12-year lease. The purchase price is $65,000.
The annual rent is $4,800 payable at the start of each year. It is estimated that
this land will be needed for only 12 years and will have a saleable value of
$80,000 at the end of the 12-year period.

With the aid of a suitable cash flow diagram, determine the rate if return,
i, that the Blue Star Company will receive by buying the land instead of leasing
it.

b) A person purchases a $6,000 automobile and makes an initial payment of


$1,500 and agrees to pay the remaining balance in 24 monthly payments of $220
each.

1) Determine the nominal interest rate involved.


2) After 6 months the person decides to increase his payments in order to
shorten his repayments period by 6 months. Assuming a nominal interest
rate of 12%. Determine the required period monthly payment during the
remaining 12 months.

Solution:

a) Cash flow diagram:

23
F=$8000

1 2 3 4 5 6 7 8 9 10 11 12

Figure (1)
P=$6500

From Figure (1):

F  P (1  i ) n  80,000
F  65,000 (1  i )12  80,000  (1)

0 1 2 3 4 5 6 7 8 9 10 11

12 yrs
……………………
A=$4,800 Figure (2)

From Figure (2):


F/A i ,n
F  A( )  A (1  i )
F/A i ,11
F  $4,800 ( )  $4,800 (1  i )  ( 2)

Solving (1) in (2):


F/A i ,11
$65,000 (1  i )12  $80,000  $4,800 ( )  $4,800 (1  i )
 (1  i ) 11
1
$65,000 (1  i )12  $4,800    $4,800 i  80,000  4,800
 i 
By trial and error:

Let i  8%
L.H .S  83398.71778

Let i  8.2%
L.H .S  86206.33042

 i between 8%  8.2%
By Interpolation:

24
86206.33042  84800 84800  83398.71778

0.082  i i  0.08

 i  0.08099
 i  8.099%  8.1%

 The rate of return i , that the blue star company will receive by buying the land
will be
i  8.1%

b)
1)
- Cash flow diagram

$1,500
A
A=$220

1 2 3 …………………… 24

P=$6,000

P/ A i ,n
P  1,500  A( )
P/ A i , 24
6,000  1,500  220 ( )

P/ A i , 24
( )  20.4545

From tables based on n=24 we can find i

 From table A.5


P/ A 1.25, 24
( )  20.6243

 From table A.6


P/ A 1.5, 24
( )  20.0304

 By interpolation:
20.6243  20.4545 20.4545  20.0304

1.25  i i  1.5
 i =1.32147%

25
 The nominal interest rate :
r  12  1.32147
r  15.86%

2)

$1,500 A0 A0

A=$220

1 2 3 … 6 7 ……………… 18

r = 12%
i = 1%

P6
P=$6,000

Let A0 equals the new A that he must pay in order to shorten his repayments by
6months.

P/ A i ,n P/ A i ,n P/F i ,n
P  1,500  A ( )  A0 ( )( )
P/ A 1,6 P/ A 1,12 P/ A 1, 6
6,000  1,500  220 ( )  A0 ( )( )
6,000  1,500  220 (5.79574)  A0 (11 .25510)(0.94205)

3224.9966
A0   $304.1626
10.6028

26
Problem 3.8:
Ms. Eiman approaches the loan tiger agency for $15,000 to be repaid in 30
monthly installments . the agency advertises interest at 1.25% per month . However ,
they proceed to calculate the size of Ms. Eiman’s payments in the following manner :
Amount requested = $15,000
Credit investigation = $60
Credit risk insurance =$95

Total = $15,155

Interest : ($15,155)(30)(0.0125) =5,683


Total owed =$20,838
$20,838
Payment : =$694.60
30
Determine the effective annual interest rate obtained by the above financing
process .

Solution:

27
A=694.60 KD

1 2 3 4 5 6 7 8 9 10 11 12 13 14 30 months

P=15,000 KD

Using linear interpolation :

P = 15,000 KD
A = 694.6 KD
n = 30
P = A ( P / A i, n )
15,000 = 694.6 ( P / A i,30 )  21.59516 = ( P / A i, n )

(1  i ) n  1
(P/ A i, n ) =
i (1  i ) n
(1  i ) 30  1
(P/ A i,30 ) =
i (1  i ) 30
(1  i ) 30  1
21.59516 =
i (1  i ) 30
This equation can be solved for (i) only numerically using the trial and error
procedure :

First trial :

Let i = 2%
(1  0.02) 30  1
= 22.395
0.02(1  0.02) 30

Second trial :

Let i = 3%
(1  0.03) 30  1
= 19.600414135
0.03(1  0.03) 30

Since the numerical value of the factor is (21.59516) it is obvious that the
required value of i lies between 2% and 3% .

28
Based on this fact we can make the following additional trial to narrow the
range .

It follows that :
i1 = 2% Y1 = 22.395

i2 = 3% Y2 = 19.60044135

i = ?% Y = 21.59516

Substituting these values into the following interpolation formula

i  i1 Y  Y1
=
i2  i1 Y2  Y1

gives:

i  0.02 0.199472
=
0.01 2.7941

i = 2.7139042%

29
Problem 3.9:
Assume that the present time is November 1985. Mr. Husam needs $300 by
the end of next month. He feels that his needs are going to increase by $30 each
month until the end of August 1986. Mr. Husam discusses the matter with the
manager of his bank who agrees to meet Mr. Husam’s monthly needs provided
that he pays 15 equal monthly payments of $315 each starting by the end of
September 1986. Assuming compounding of interest, determine the nominal
interest rate involved in this financial agreement.

Solution:

Cash flow diagram

30
A0 = $300
10 11 12 13 14 15 16 17 18 19 20 21 22 23 24 months
0 1 2 3 4 5 6 7 8 9
1985

A = $315
A0 = $300
G = $30
r =??

The two series are equivalent to each other, they have the same present worth at
the same time, let this time be November 1985.

P1 = P2

 A / G i ,n P / A i , n P / A i , n F / P i , n
[A0 + G   ]*   =A     *

      

 A / G i , 9  P / A i , 9  P / A i, 15 F / P i , 9
[$300 + $30   ]*   = $315     *

      

31
 A / G i , 9  P / A i , 9  P / A i, 15 F / P i , 9
[$300 + $30   ]*   - $315     * = 0.0

      
Using trial and error to find i:

First: let i = 1%

 A / G i , 9  P / A i , 9  P / A i, 15  F / P i , 9 
[$300 + $30   ]*   - $315     * = - 412.67421

 3.93367   8.56602   13.8650   0.914340


Second: let i = 1.75%

 A / G 1.75 , 9  P / A 1.75 , 9  P / A 1.75 , 15 F / P 1.75,9


[$300 + $30   ]*   - $315    
* =

 3.88438   8.26052   13.0929   0.85544 


- 87.294

Third: let i = 2%

32
 A / G 2 , 9  P / A 2 ,9  P / A 2 , 15  F / P 2 , 9
[$300 + $30   ]*   - $315     * = 9.0295

 3.86805   8.16222   12.8492   0.83676 


By linear interpolation:

At: i1 = 1.75% x1= - 87.294

i =? x=0
i2 = 2% x2 = 9.0295

(x-x1)/ (i-i1) = (x2-x1)/ (i2-i1)

(0+87.294)/ (i-0.0175) = (9.0295+87.294)/ (0.0025)

i = 1.976564 %

r = i*c = (1.976564 %)*12 = 23.71876%

c
i (eff) = (1+r/c) -1 = (1+0.01976564)12 -1 = 0.264749

i (eff) = 26.4749% per year

Factors formulas:

33
 A / G i ,n
  n
= (1/i) – n / ((1+i) – 1)

 

P / A i , n
  n n
= ((1+i) - 1)/ (i*(1+i) )

 

F / P i , n
  = (1+i)
n

 

34
Problem 3.10:
The White Hand Trust Company offers the following personal loan plan
called the "One Percent Plan". For an 18 months payment period, the plan
operates as follows:
The company adds 18th% (1% for each month) to the amount borrowed, the
borrower pays back one-18 of this total at the end of each month for 18 months.
The borrower also has to pay 0.5% of the total loan as a service charge as soon as
he receives the loan.
Sketch the cash flow diagram for this plan and find its nominal and effective
rates per annum.

Solution:

Cash flow diagram:

(1/18 + 0.01)P
0.05 P

0 1 2 3 4 5 18

35
 P/Ai,n
P=A   + A0

 
 P/Ai,18
P = (1/18 +0.01) P   + 0.05 P

 
 P/Ai,18
1 = (1/18 +0.01)   + 0.05

 
 P/Ai,18
0.95 = 0.0655  
 
 P/Ai,18
  = 0.95 / 0.0655 = 14.504

 
From interest tables (Table A.8 & Table A.9):

36
 P/A2,18
  = 14.992

 
 P/A3,18
  = 13.7535

 
To find i we need to use linear interpolation:

(i-0.02)/ (14.992-14.504) = (0.03-0.02)/ (14.992-13.7535)

i= 0.02394 = 2.394 %

Nominal interest rate = i * 12= 28.758%

Effective interest rate = ((1+ i) 12 -1) = 32.829 %

Linear interpolation

37
14.9920

14.5040

13.7535

2 i 3

Problem 3.12:
a ) determine the present equivalent value of $1500 paid every 3 month over a
period of 5 years in each of these situation :
1) Nominal interest rate of 18% compounded annually.

38
2) Nominal interest rate of 18% compounded quarterly.
3) Nominal interest rate of 18% compounded monthly.
4) Nominal interest rate of 18% compounded continuously.

b-) what equal series of payment are necessary to repay a loan of $45000 inn 5
years at nominal interest rate of 15% using the following payment and compounding
frequencies :
1) Monthly payments with monthly compounding.
2) Annual payments with semiannual compounding.
3) Semiannual payments with annual compounding.
______________________________________________

a)
i) compounded annually:-

Fc = 1< Fp = 4

i = 18% /1 = 18%

- using the w-shifting method , the equivalent cash-flow diagram :

A1 = 1500(F∕A i,n) = 1500 (F∕A 18,5) = $6417.3


P = A1 (P/A 18,5)
= 6417.3 (3.12717)
= $20068.98

39
ii) compounded quarterly :-

Fc = 4 = Fp

i = 18% /4 = 4.5% , n = 20

P = A(P/A i,n) = 1500 (P/A 4.5,20) = 1500 (1+i)n−1


i(1+i)n

= $1500 ( 13.00793635) = $19511.904

- if we use interpolation which is less accurate :

(P/A 4,20) and (P/A 5,20) then (P/A 4.5,20)

(13.59030) and (12.46220) then (13.02625)

P = $1500(P/A 4.5,20) = $ 1500(13.02625)


= $19539.375

ii) compounded monthly

Fc = 12 < Fp = 4

approach (1) :

ieff = ( 1 + 18/12)3 − 1 = 4.56784%

P = A(P/A i,n) = $1500 (P/A 4.5,20) = $1500 (1+i)n−1


i(1+i)n
= $1500 x 12.93180632
= $ 19397.71

40
approach (2) :

A = F (A/F i,n)
= $1500( A/F 1.5, 3)
= $1500 (39.38020)
= $10397.51

iv) compounded continuously :

41
Fc = ∞ > Fp = 4

P = A[P/A i,n] = $1500 [P/A 4.5,20]

= $1500 1− e−rn
er − 1

= $1500 x 12.89283977 = $ 19339.259

b)
i) monthly payments with monthly compounding :

Fc = 12 = Fp

A = P(A/P i,n) = $45,000(A/P 1.25,60)

= $45,000(0.0238) = $10711

ii) annual payments with semi-annual compounding :-

Fc = 2 > Fp = 1

42
ieff = (1 + 15/2 ) 2 −1 = 15.5625 %

A = P(A/P i,n) = $45,000 (A/P 15.5625, 5)

= (1+i)n−1 = $45,000 x 0.3022298294


i(1+i)n

= $ 13603.42

iii) semi-annual payments with annual compounding :-

Fc = 1 < Fp = 2

43
using the w-shifting equivalent cash-flow diagram :

2A = P( A/P i,n)
2A = $45,000 ( A/P 15,5) = $45,000 (0.29832)

A = $6712.2

44
Problem 3.13:
a) Miss Nuha plans to deposit $200 weekly into a savings account starting from the
8th of March until the end of May .determine the balance in Miss. Nuha's
account at the end of May if the account pays a nominal interest rate of 10%
compounded:
1) Weekly.
2) Daily.
3) Monthly.

b) Mr. Mahir makes quarterly deposits of $750 into an account which pays a
nominal interest rate of 18 %. Determine the account balance immediately after
the twelth deposit if the internal rate is compounded with following frequency,
fc:
1) fc = 2
2) fc = 4
3) fc = 12
4) fc = 2 for first year, fc = 4 for the second year ,fc = 12 for the third year of
the loan.

Solution:
a) Cash flow diagram

1) Compound weekly :
fc = fp , iweekly = r / 52 = 0.1923%

45
F / A i , n F / P i, n
F  A( )  A( )
 (1  i ) n  1 
F  A  (1  i ) n 
 i 
 (1  0.001923)  1 12

F  $200  (1  0.001923)12 
 0.001923 
F  $20013.1511 = $ 2630.21256

2) Compound daily:
fc > fp , idaily = r / 365 = 0.0273972 %
ieff  (1  idaily ) 7  1
ieff  (1  .000273972) 7  1  0.1919385%
From (1):
 (1  0.001919385)12  1 
F  $200  (1  0.001919385)12 
 0.001919385 
F  $20013.15077  = $ 2630.15415

3) Compound monthly:
fc < fp
By using W_D shifting method:

i = r/12 =.1/12= 0.8333% , n=3

46
F / A i, n F / P i , n
F  4A( )  A( )
 (1  i ) n  1 
F  A 4  (1  i ) n 
 i 
 (1  .008333)  1 3

F  $200 4  (1  0.008333) 3 
 .008333 
F  $20012.100276  1.0252089 = $ 2625.097338

b) Cash flow diagram

1) fc=2 (compound semi-annually) fc < fp


By using W_D shifting method:

47
i = r/2 =18% / 2 = 9 %, n=6

F / A i, n
F  2 A( )
 (1  i ) n  1
F  2 A 
 i 
 (1  0.09) 6  1
F  $1,500   = $ 11285.00185
 0.09 

2) fc = 4 , fc = fp.
i = r/4 = 4.5 % & n=12

F / A i, n
F  A( )
 (1  0.045)12  1
F  $750  = $ 11598.02388
 0.045 

3) fc=12 , fc > fp
imonthly = r/12 = 1.5%
ieff ,quartely  (1  imonthly ) 3  1 = 4.56783%

F / A ieff , n
F  A( )
 (1  0.0456783)12  1
F  $750   = $ 11643.467
 0.0456783 

4) Cash flow diagram

48
Take the first years: fc = 2, fc < fp
By using W_D shifting
i = r / 2 = 9 %, n=2

F / A i, n
F1  2 A( )
 (1  0.09) 2  1
F1  $1,500   = $ 3135
 0.09 
Take the second years: fc = fp = 4
i = r/4 = 4.5 %, n = 4

F / A i, n
F2  A( )
 (1  0.045) 4  1
F2  $750   = $ 3208.643344
 0.045 
Take the third years: fc = 12 , fc > fp
imonthly = r/12 = 1.5%
ieff ,quartely  (1  imonthly ) 3  1 = 4.56783%

F / A ieff , n
F3  A( )
 (1  0.045678) 4  1
F3  $750  = $ 3211.881919
 0.045678 

49
Let:
i2 : annually interest rate during second year.
I3 : annually interest rate during third year.
r 0.18 4
i2,annually  (1  ) 4  1  (1  )  1  19.2518%
4 4
r 0.18 12
i3,annually  (1  )12  1  (1  )  1  19.5618%
12 12

 F / iP 2,1  F / iP 3,1
F  F1( ) F2( ) F3
 
 
F  $3135(1  i2 )1  $3208.643344 (1  i3 )1  $3211 .881919
F   $3738.5439  $3208.643344 (1.195618)  $3211.881919
F  $8306.182149  $3211.881919
= $ 11518.06407

Problem 3.13:
a) Miss. Nuha plans to deposit $200 weekly into saving account starting from the
8th of March until the end of May. Determine the balance in Miss. Nuha's account
at the end of May if the account pays a nominal interest rate of 10%
compounded:
i) Weekly.
ii) Daily.
iii) Monthly.

b) Mr. Mahir makes quarterly deposits of $750 into an account which pays a
nominal interest of 18%. Determine the account balance immediately after the
twelfth deposit if the internal rate is compounded with the following, fc:
i) fc = 2 ii) fc = 4 iii) fc =
12
iv) fc= 2 for the first year, fc = 4 for the second year and fc = 12 for the third
year of the loan.

Solution:

50
Part a:-

i) fc = fp (payments period is equal to compounding frequency)

i= 10 = 0.1923
52
F = A (F/A i, 12)

F=A (1+i)ⁿ-1
i
F = A*12.12822535 =200*12.12822535 = $2425.65

ii) fc = 7 , fp = 1
Fc>fp (compounding more frequent than payments)

1st approach:-

i = 10 = 0.0274%
365
C= fp\fc = 7 / 1 = 7 (the number of actual compounding periods per payment
period)
ieff = (1+i)^c -1
ieff = (1+0.000274)^7 -1 = 0.19195%
F = A ( F/A i ,12)
F = A*12.138 = 200*12.1267 = $ 2425.35

2nd approach:-

51
i = 10 = 0.0274%
365
A = F (A/F i, 7)
A = 20(A/F 0.0274, 7 )
A = F*[i / (1+i) ⁿ - 1]

A = F* 0.14274 = 200*0.14274 = 28.549

F = A (1+i)ⁿ -1
i

F = A*84.95 = 28.549*84.95 = $ 2425.59

iii) 1st approach :-

W-D shifting method:-

52
A* = 4*200 = 800
i = 10 / 12 = 0.8333%

F = A* (1+i)ⁿ_1
i
F = (A*)*3.025 06 = $2420.055

2nd approach:-

Minimum-Balance Method:-

Basis: compounding period = 1 month


n = 3 periods
Payments period = 12
i = 10/12 = 0.8333%

End of n Bs D Bn Bm I

53
week interest Starting deposits Net Minimum Interest
period Balance balance Balance Earned
($) ($) ($) ($)
0 0.0 0.0
1 200 200
2 1 200 400 0.0
3 200 600
4 200 800 0.0
4 800 800
5 200 1000
6 2 200 1200 800
7 200 1400
8 200 1600 6.664
8 1606.66 1606.66
4 4
9 200 1806.66
4
10 3 200 2006.66 1606.664
4
11 200 2206.66
4
12 200 2406.66 13.385
4

F3 = B*n-3 + I3 = 2406.664 + 13.385 =$ 2420.0475

54
b)

i) fc = 2 , fp = 4 (quarterly payments with interest compounded semi-annually)


fc< fp

1ST approach:-
W-D shifting method:

55
i = 18 = 9%
2
F = A* (F/A i, n)

A*=2*$750 = $1500

F = A* (1+i)ⁿ-1
i
F = $1500*7.523 = $11285

2ND approach :-

Minimum Balance Method:

End of n Bs D Bn Bm I
3 interest Starting $Deposit Net Minimum Interest
months period Balance balance Balance Earned
($) ($) ($) ($)
0 0.0 0.0
1 1 750 750 0.0
2 750 1500 0.0
2 1500 1500
3 2 750 2250 1500
4 750 3000 135
4 3135 3135
5 3 750 3885 3135
6 750 4635 282.1
5
6 4917.15 4917.15
7 4 750 5667.15 4917.15
8 750 6417.15 442.5
4
8 6859.69 6859.69
9 5 750 7609.69 6859.69
10 750 8359.69 17.37
1
10 8977.062 8977.06
1 2
11 6 750 9727.06 8977.06
2
12 750 10477.0 807.9

56
6 3

F6 = B*n-3+I3 = 10477.0621+807.935 = $11284.997 ≈ $11285


ii) fc = 4, fp = 4 (quarterly payments with interst compounded quarterly)
fc = fp

F = A (F/A i,n
)

F = A* (1+i)ⁿ-1
i

F = A*15.464 = 750*15.464 = $11598.0

iii)fc = 12, fp = 4
fp>fc (compounding more frequent than payments)
1st approach:
i = 18 = 1.5%
12
ieff = (1+.015)^3_1 = 4.5678%

F = A (F/A i,n
)

F = A (1+i)ⁿ-1
i
F = A*15.52475 = $11643.56

57
2ND approach:-

A = F* i (1+i)ⁿ
-1

F = A ( F/A i ,n )

F = A (1+i)ⁿ-1
i
n= 36, i= 1.5%
F = A*47.275969 = 246.287*47.275969 = $11643.456

58
-fc = 2 for 1 year
W-D shift method:

F1 = A*(F/A i, n
)

F= A (1+i)ⁿ-1
i

i = 18/2 = 9%
A*= 2*$750 = $1500
n= 2
F1 =( A*)*2.09 = $3135
-fc = 4 for second year

F2 = A(F/A i,n
)

F = A (1+i)ⁿ-1
i
i = 18/4 = 4.5%
n=4
F2 = A*4.278 = 750*4.278 = $3208.6433

fc = 12 for the third year


Using first approach in part (ii):

F3 = A (F/A i,n)
i = 18/12 = 1.5%
ieff = (1+.015)^3_1 = 4.5678%
F= A (1+i)ⁿ-1
i

59
F3 = A*4.2825 = 750*4.2825 = $3211.91

Ftotal = F3 + F2 (F/P i,n) + F1(F/P i,n)

Ftotal = $3211.91+$3208.6433 (F/P 18,1 ) +$3135(F/P 18,2)

Ftotal = $3211.91+$3208.6433*1.18 +$3135*1.3924


Ftotal = $11363.283

Problem 3.14:
The Mercy Savings and Loan Association advertises that it pays 16%
interest compounded quarterly.

a) If Mr. Husam deposits $ 9000 now and plans to withdraw it after five
years, how much would his account be worth at that time?
b) If Mr. Husam decides to deposits $ 18000 every year for 5 years,
determine the amount accumulated in his account at the end of the fifth
year.
c) Suppose that Mr. Husam makes monthly deposits of $ 150 for the 5-
year period. How much could be withdrawn at the end of the period.

Solution:
i = 16% compounding quarterly
fc = 4

a) p = $ 9000 , n = 5 years , f=?


Cash flow diagram is below:

F=??

0 1 2 3 4 5
P=9000$ i =16%

60
i = 16/4 = 4% every three-month period
compounding period = 3-month period
number of interest period, n = 5*4 = 20 periods
interest rate per period, i = 4%
f/p 4,20

f=p( ) = 9000 * (2.19112) = $ 19720.08

b) annual payment A = $ 1800


n = 5 years
adopting the payment period as the basis for calculation :

ieff = (1+ i)c - 1


where:
i = interest rate per original compounding period which is three month period.
= 16/4 = 4%
c = number of compounding periods per effective period (one year)
c = one year period per/3-month period = 12/3 = 4
ieff = (1+4/100)4 -1 = 16.985%
Cash flow diagram is below:

0 1 2 3 4 5 years

A=1800 A A A

i = 16.985% annually

f/A i,n
F=A ( )
Using interest tables:
F = 1800 * 7.01232 = $ 12622.18074

c) A = $150 monthly , n = 5 years


we have three periods each period is with three payments.

Approach I :
Using W-D shifting method:

61
0 1 year
A A A

Cash flow diagram is below:

F = ??

1 2 3 4 5
20

3A 3A 3A

3A = 3* 150 = $450
F = A (f/A 4,20) = 450 * 29.7787
= $13400.415

Approach II:
Using minimum balance approach
Month of Interest Bs D Bn net Bm min Ij interest earned
year period starting deposits balance balance per period j ($)
n balance ($) ($) ($)
($)
0 1 0 0 0 0 0
1 150 150
2 150 300
3 150 450
3 2 450 450 450 I2 = 450 * (4/100)
4 150 600 = 18
5 150 750
6 150 900
6 3 918 918 918 36.72
7 150 1068
8 150 1218
9 150 1368
9 4 1404.72 1404.72 1404.72 56.1888
10 150 1554.72
11 150 1704.72
12 150 1854.72

62
12 5 1910.9 1910.9 1910.9 76.44
13 150 2060.9
14 150 2210.9
15 150 1260.9
15 6 2437.34 2437.34 2437.34 94.49
16 150 2587.34
17 150 2737.34
18 150 2887.34
18 7 2984.83 2984.83 2984.83 119.39
19 150 3134.83
20 150 3284.83
21 150 3434.83
21 8 3554.22 3554.22 3554.22 142.17
22 150 3704.22
23 150 3854.22
24 150 4004.22
24 9 4146.39 4146.39 4146.39 165.86
25 150 4296.39
26 150 4446.39
27 150 4596.39
27 10 4762.25 4762.25 4762.25 190.49
28 150 4912.25
29 150 5062.25
30 150 5212.25
30 11 5402.74 5402.74 5402.74 216.11
31 150 5552.74
32 150 5702.74
33 150 5852.74
33 12 6068.85 6068.85 6068.85 242.75
34 150 6218.85
35 150 6368.85
36 150 6518.85
36 13 6761.6 6761.6 6761.6 270.46
37 150 6911.6
38 150 7061.6
39 150 7211.6
39 14 7482.06 7482.06 7482.06 299.28
40 150 7632.06
41 150 7782.06
42 150 7932.06
42 15 8231.34 8231.34 8231.34 329.25
43 150 8381.34
44 150 8531.34
45 150 8681.34
45 16 9010.59 9010.59 9010.59 360.42
46 150 9160.59
47 150 9310.59
48 150 9460.59

63
48 17 9821.01 9821.01 9821.01 392.84
49 150 9971.01
50 150 10121.01
51 150 10271.01
51 18 10663.85 10663.85 10663.85 426.55
52 150 10813.85
53 150 10963.85
54 150 11113.85
54 19 11540.4 11540.4 11540.4 461.62
55 150 11690.4
56 150 11840.4
57 150 11990.4
57 20 12452.02 12452.02 12452.02 498.08
58 150 12602.02
59 150 12752.02
60 150 12902.02
13400.1

F = $1 3400.1

64
Problem 3.15:
An engineer is planning to retire in 30 years. He wishes to deposit a
regular amount every 3 months until he retires so that beginning one year after
his retirement, he will receive annual payment of $ 10,000 for the next 20 years.
How much must he deposit every 3-month period assuming that the nominal
interest rate is 8% compounded:
a) Quarterly?
b) Monthly?
c)Annually?

Solution

a) r = 8% compounded quarterly:

65
ieff-annually= (1+r/c)c – 1 where r = 8%, c=4
4
ieff-annually= (1+0.08/4) – 1= 8.243%

P*= Aw(P/A i,n)=


P*=10,000(P/A 8.243,20) =10,000[(1+i)n-1/i(1+i)n]
=10,000[(1+0.08243)20-
1/0.08243(1+0.08243)20]=10,000*9.64311=$96,431.16

F*=P*
Ad=F*(A/F i,n) where i=%8/4=%2 & n=30*4=120
=96,431.16(A/F 2,120)=96,431.16[i/(1+i)n-1]= 96,431.16[0.02/(1+0.02)120-1]
=96,431.16*0.002048=$197.5
b) Compounded Monthly:

ieff-annually= (1+r/c)c – 1 where r = 8%, c=12


ieff-annually= (1+0.08/12)12 – 1= 8.299%

P*= Aw(P/A i,n)=


P*=10,000(P/A 8.299,20) =10,000[(1+i)n-1/i(1+i)n]
=10,000[(1+0.08299)20-
1/0.08299(1+0.08299)20]=10,000*9.6035=$96,034.82

F*=P*
ieff-quarterly= (1+r/c)m – 1 where r = 8%, c=12 m=3
ieff-quarterly= (1+0.08/12)3 – 1= 2.0134%

Ad=F*(A/F i,n) where i= ieff-quarterly & n=30*4=120


=96,034.82 (A/F 2.0134,120)= 96,034.82 [i/(1+i)n-1]
= 96,034.82 [0.020134/(1+0.020134)120-1]
=96,034.82*0.002026=$194.598

66
c) Compounded Annually:

P*= Aw(P/A i,n)=


P*=10,000(P/A 8,20)
=10,000[(1+i)n-1/i(1+i)n]
=10,000[(1+0.08)20-
1/0.08(1+0.08)20]=10,000*9.8
18=$98,181.47

F*=P*
Using W-S shifting technique
A*=4Ad
A*=F*(A/F i,n) where i=8% & n=30
=98,181.47 (A/F 8,30)= 98,181.47 [i/(1+i)n-1]
= 98,181.47 [0.08/(1+0.08)30-1]
=98,181.47*0.008827=$866.69
Ad=A*/4=$216.6724

Comments on the original solution:


 In a) answer difference is due to round-ff error in ieff
 In b) there are errors in calculating the equal-payment series
present worth factor (P/A 8.299,20)
 Modifications in cash flow diagrams are carried out.
 In general as the compounding frequency decreases the sinking-
fund (A) increases.

67
Problem 3.16:
An increasing annual uniform gradient series begins at the end of the third
year & continues until the end of the twelfth year. What is the value of the
gradient ,G, that makes the gradient series equivalent to a series of equal
end-of -month payments of $800 for 5 years followed by end-of –quarter
payments of $2500 foe 3 more years at a nominal interest rate of 16%
compounded quarterly?

The equivalent cash flow diagram for the problem is as follows:-

Nominal interest rate r = 16%


i=16%/4= 4%
Period of comparison ,n = 12years=12×4=48 quarters
$800 monthly = 3×$800 =$2400 quarterly
n1= 5 years= 5×4=20 quarters
F1 = 2400(F/A 4,20)(F/P 4,28) = 2400×29.77781×2.9987= $214309.64
n2=3 years=3×4=12 quarters
F2 = 2500(F/A 4,12) (F/P 4,16)= 2500×15.258×1.8730 = $70357.6135
FT = F1 + F2 = $284667.2535

68
For the gradient series:-
effective interest rate = ieff=(1+0.04) 4 -1 ≈ 17%
A= Ao+G(A/G 17,10), but A= G
A/G 17,10
A= G (1+( ))
FT = A (F/A 17,10)= G (1 + (1+(A/G 17,10))× (F/A 17,10) = G×4.2555×22.393
G=$298.73

69
Problem 3.17:
Ms .Eiman borrows $10000 and repays the loan in five equal annual payments.
The interest rate for the first year of the loan is 8% compounded annually for the
second year is 10% compounded annually and for the third year is12%
compounded annually.
However, for the fourth and fifth year quarterly and monthly , respectively .
Determine the size of the equal annual payments made by Ms. Eiman.

Solution

i1=8% i2=10% i3=12%

i4=12% compounded quarterly


i4=12%/4 =3%

ieff4 = (1+ i4) n


ieff4= (1+0.03)4 -1 =0.12551 =12.551% annually

70
i5=12%/12=1%

ieff5=(1+0.01)12-1=0.126825=12.6825% annually
F/P I,n
F1=P( ) = $10000(1.08000)

F1=$10800.

F/P i,n
F2=F11( )=$10800(1.10000)

F2=$11880
F/P i,n
F3=F2( )=$11880(1.12000)

F3=$13305.6
(F/P i,n
F4=F3 )=$13305.6(1.12551)

F4=14975.58585
F/P i n
F5=F4( )=$14975.58585(1.126825)

F5=$16874.86452

Calculated i :
F/P i,n
F5=P ( )
5
16874.86452 = $10,000*(1+i)
5
(1+i) = 1.6874
5ln(1+i) = ln(1.6874)
Ln(1+i)= 0.1046
0.1046
1+i = e
i = 11.03197%

Calculated annual payment (A)

A/F i,n
A=F5 ( )
A/F i,n 5
( ) = 0.1103197 / [(1+0.1103197) -1]
=623.762776
A =16874.86452 * 623.762776
A =$2707.879

71
72

Anda mungkin juga menyukai